Đến nội dung

minhtuyb nội dung

Có 497 mục bởi minhtuyb (Tìm giới hạn từ 30-03-2020)



Sắp theo                Sắp xếp  

#326959 Cách giải phương trình bậc 4 bằng máy tính

Đã gửi bởi minhtuyb on 19-06-2012 - 11:01 trong Kinh nghiệm học toán

Đây là một số bài dễ, áp dụng như trên là được !!!
(Cẩn thận bị lừa)
a) $x^4+x^3+x^2+2=0$

Chưa quen tay, mãi mới làm đc 1 con :(
Đặt $x=y-\frac{1}{4}$ thì
$$x^4+x^3+x^2+2=(y-\frac{1}{4})^4+(y-\frac{1}{4})^3+(y-\frac{1}{4})^2+2\\ =y^4+\frac{5}{8}y^2-\frac{3}{8}y+\frac{525}{256}\\ =y^4+\frac{5}{8}(y-\frac{3}{10})^2+\frac{2553}{1280}>0$$
Vậy pt đã cho vô nghiệm :icon6:
---
P/s: Chọn $m=0$ :P



#326952 Cách giải phương trình bậc 4 bằng máy tính

Đã gửi bởi minhtuyb on 19-06-2012 - 10:47 trong Kinh nghiệm học toán

Ta sẽ chứng minh $f(x)>0$ bằng cách đặt $x=y-\frac{a}{4}$, để mất đi hệ số của $y^3$

Đặt $x=y+\frac{3}{2}$

Biểu thức đã cho trở thành:
$$A=y^4+\frac{5y^2}{2}-y+\frac{61}{16}=y^4-m y^2+m^2+(m+\frac{5}{2})y^2-y+\frac{61}{16}-m^2$$

Mình cũng chia sẻ chút chỗ này ^_^:
Khi đã ra $A=y^4+\frac{5y^2}{2}-y+\frac{61}{16}$ thì trước khi chọn hệ số $m$ thích hợp như trên nên kiểm tra xem tam thức bậc hai $\frac{5y^2}{2}-y+\frac{61}{16}$ có vô nghiệm hay không:
+) Nếu vô nghiệm $(\Delta <0)$ thì ta phân tích thẳng luôn: $A=y^4+\frac{5}{2}(y-\frac{1}{5})^2+\frac{297}{80}$, tức là chọn $m=0$ để đỡ mất công cho phần sau
+) Nếu có nghiệm thì lại phải lục cục đi tìm $m$ thôi :P
---
P/s: Chú kia trích dẫn ngắn thôi, mất thẩm mĩ ="='



#318269 Topic luyện thi vào lớp 10 năm 2013 – 2014 (Hình học)

Đã gửi bởi minhtuyb on 21-05-2012 - 14:18 trong Hình học

Bài 78: Cho nửa đường tròn tâm O đường kính AB và tiếp tuyến Ax cùng phía với nửa đường tròn đối với AB. Từ M trên Ax kẻ tiếp tuyến thứ hai MC với đường tròn( C là tiếp điểm). $AC\cap OM=\left \{ E \right \}; MB\cap (O)=\left \{ D \right \}$ (D #B)
a. C/m: tứ giác AMCO nội tiếp;
b. $\widehat{ADE}=\widehat{ACO}$;
c. $CH\perp AB (H\in AB)$. C/m: MB đi qua trung điểm CH.

Chọn lọc bài để post nhé bạn. Bài này có nhiều rồi! ^_^:
Hình đã gửi
$b)\widehat{ADE}=\widehat{AME}=\widehat{ACO}$
c) - Tia $BC$ và $AM$ kéo dài cắt nhau tại $F$. Có $\Delta AMC$ vuông; $AM=MC\Rightarrow AM=MC=MF$ (nếu muốn chắc chắn thì bạn c/m $\Delta FMC$ cân rồi suy ra điều này)
Mặt khác theo Thales dễ dàng cm:
$$\frac{CI}{IH}=\frac{MF}{AM}=1\Rightarrow CI=IH\ <Q.E.D>$$



#372525 BĐT AM-GM

Đã gửi bởi minhtuyb on 25-11-2012 - 17:31 trong Bất đẳng thức và cực trị

a,Hãy suy xét một bài toán cơ bản
Ta xét BĐT sau:Với mọi a,b,c dương có tổng bằng 3
$\frac{1}{a^2(1+a)}+\frac{1}{b^2(1+b)}+\frac{1}{c^2(1+c)}\geq \frac{3}{4abc}$(Olimpic 30-4)
Giải:Ta hoàn toàn có thể giải quyết bài toán trên bằng đánh giá thuận C-S,tuy nhiên,ta sẽ xem xét 1 lời giải sau
BĐT
$\Leftrightarrow \frac{abc}{a^2(1+a)}+\frac{abc}{b^2(1+b)}+\frac{abc}{c^2(1+c)}\geq \frac{3}{4}$
$\Leftrightarrow \frac{bc(b+c)}{a\left [ (b+c)^2+2a(b+c) \right ]} +\frac{ca(c+a)}{b\left [ (c+a)^2+2b(c+a) \right ]}+\frac{ab(a+b)}{c\left [ (a+b)^2+2c(a+b) \right ]}\geq \frac{3}{4}$
$\Leftrightarrow \frac{bc(b+c)}{2a\left [ (b^2+c^2+a(b+c) \right ]} +\frac{ca(c+a)}{2b\left [ a^2+c^2+b(c+a) \right ]}+\frac{ab(a+b)}{2c\left [ a^2+b^2+c(a+b) \right ]}\geq \frac{3}{4}$
$\Leftrightarrow \frac{(b+c)}{2a(\frac{a+b}{c}+\frac{a+c}{b})} +\frac{(c+a)}{2b(\frac{b+c}{a}+\frac{a+b}{c})}+\frac{(a+b)}{2c(\frac{b+c}{a}+\frac{c+a}{b})}\geq \frac{3}{4}$

Bằng việc đặt $\frac{a+b}{c}=x,\frac{b+c}{a}=y,\frac{c+a}{b}=z$,ta dễ dàng đưa BĐT trên về nesbitt 3 biến

Yêu cầu cậu đặt lại phép suy luận toán học ở bài này! Có chỗ là dấu $\Leftarrow$ chứ không phải $\Leftrightarrow$.

c,Làm mạnh BĐT

Tư tưởng giải 2 bài này giống như bài trên nên các bạn luyện tập thêm
"Cũng chẳng có gì khó cả,cái này bình thường thôi"-tôi tin là bạn đang nghĩ vậy,tuy nhiên,bạn hãy xem thêm 2 VD sau để chắc chắn cái bạn nói là đúng nhé
Từ BĐT AM-GM 3 số,ta có thể làm mạnh lên thành
$\frac{a+b+c}{3}\geq \sqrt[3]{abc}+\frac{(a-b)^2+(b-c)^2+(c-a)^2}{12(a+b+c)}$
Cũng cần nói thêm là BĐT này cần tới S.O.S để giải (quá ảo) :lol:

Chưa thử c/m BĐT này bằng $S.O.S$. Nhưng mình cũng xin giới thiệu một BĐT khác cũng khá giống cái này, nhưng được làm mạnh trực tiếp từ $AM-GM$:
$\dfrac{a+b+c}{3}\geq \sqrt[3]{abc}+\dfrac{(\sqrt{a}-\sqrt{b})^2+(\sqrt{b}-\sqrt{c})^2+(\sqrt{c}-\sqrt{a})^2}{3}$

Qủa này là hệ số bất định phải không minhtuyb hầy? :wacko:

Cũng không hẳn. Mình dùng pp tiếp tuyến thôi cho nhanh (bấm máy ra luôn đạo hàm tại $x=1$ là $0$ ) :D



#372298 BĐT AM-GM

Đã gửi bởi minhtuyb on 25-11-2012 - 00:17 trong Bất đẳng thức và cực trị

Mình xin đóng góp cho topic bài này
Bài 5: Cho $a,b,c>0$ và $a+b+c=3$. CMR
$\sum \frac{x}{x^4+y+z}\geq 1$

Mình lại c/m được $\sum \frac{x}{x^4+y+z}\leq 1\ (*)$ Hình đã gửi
---
Vời giả thiết trên thì: $\dfrac{x}{x^4+y+z}=\dfrac{x}{x^4+3-x}$
Ta sẽ c/m: $\dfrac{x}{x^4+3-x}\le \dfrac{1}{3}\Leftrightarrow x^4-4x+3\ge 0\Leftrightarrow (x-1)^2(x^2+2x+3)\ge 0$ (Luôn đúng)
C/m tt rồi cộng lại ta suy ra $(*)$ đúng.
Dấu bằng xảy ra khi $x=y=z=1\ \square$
---
Thôi đi ngủ đây, lần sau bạn nhớ post bài nào liên quan tới topic nhé Hình đã gửi



#372296 BĐT AM-GM

Đã gửi bởi minhtuyb on 25-11-2012 - 00:09 trong Bất đẳng thức và cực trị

c,Đặt ẩn phụ
Vì kĩ thuật đặt ăn phụ mình đã trình bày nên xin chỉ nêu 1 VD nho nhỏ sau
Chứng mnih với mọi a,b,c dương,ta có
$\frac{1}{a\sqrt{a+b}}+\frac{1}{b\sqrt{b+c}}+\frac{1}{c\sqrt{c+a}}\geq \frac{3}{\sqrt{2abc}}$
Giải BĐT cần chứng minh tương đương $\sqrt{\frac{2bc}{a(a+b)}}+\sqrt{\frac{2ca}{b(b+c)}}+\sqrt{\frac{2ab}{c(c+a)}}\geq 3$
Ta sẽ đặt $\sqrt{\frac{2bc}{a(a+b)}}=x;\sqrt{\frac{2ca}{b(b+c)}}=y;\sqrt{\frac{2ab}{c(c+a)}}=z$
Chú ý thêm rằng xy=$\frac{2c}{\sqrt{(a+b)(b+c)}}$,ta sẽ chứng minh $xy+yz+zx\geq 3$
Tiếp tục đặt $\sqrt{a+b}=p,\sqrt{b+c}=q,\sqrt{c+a}=r$,khi đó,dề thấy $xy=\frac{p^2+q^2-r^2}{qr}$
BĐT cần chứng minh sẽ là $(p^3+q^3+r^3)(p^2q+q^2r+r^2p)\geq q^2p+r^2q+p^2r+3pqr$(Đây là 1 BĐT khá hay,xin nhường cho các bạn)

Hix $p,q,r$ trâu bò quá ="=. Mình xin đóng góp lời giải theo mình nghĩ là hay hơn :lol: :
---
BĐT cần chứng minh tương đương:
$$\sum \sqrt{\frac{2bc}{a(a+b)}}\geq 3\ (*)$$
Ta có:
$$LHS(*)=\sum \dfrac{2bc}{\sqrt{2abc(a+b)}}=\sum \dfrac{2bc}{\sqrt{2ab(ac+bc)}}\ge \sum \dfrac{4bc}{2ab+ac+bc}$$
Đặt $x=ab;y=bc;z=ca\Rightarrow x,y,z>0$. Ta cần c/m:
$$\sum \dfrac{4x}{x+2y+z}\ge 3\Leftrightarrow \sum \dfrac{x}{x+2y+z}\ge \dfrac{3}{4}\ (**)$$

(Đến đây nếu dùng $Schwarz$ thì ra luôn. Nhưng vì là topic $AM-GM$ nên ta xài luôn $AM-GM$ :icon6: )

Chuẩn hoá: $x+y+z=3$. Viết lại $(**)$:
$$\sum \dfrac{x}{y+3}\ge \dfrac{3}{4}$$
Đúng từ các BĐT sau:
$$\dfrac{x}{y+3}+\dfrac{x(y+3)}{16}\ge \dfrac{x}{2}\\ \dfrac{y}{z+3}+\dfrac{y(z+3)}{16}\ge \dfrac{y}{2}\\\dfrac{z}{x+3}+\dfrac{z(x+3)}{16}\ge \dfrac{z}{2}\\ \dfrac{(x+y+z)^2}{3.16}\ge \dfrac{xy+yz+zx}{16}$$
Vậy $(**)$ đúng $\Rightarrow (*)$ đúng.
Dấu bằng xảy ra khi $a=b=c$
Phép c/m hoàn tất $\square$.



#372270 BĐT AM-GM

Đã gửi bởi minhtuyb on 24-11-2012 - 23:06 trong Bất đẳng thức và cực trị

Nhìn phát thấy ngay là $Holder$ Hình đã gửi
---
-Nếu $\exists a_i+b_i=0\Rightarrow a_i=b_i=0\Rightarrow VT(*)=VP(*)$
-Nếu các biến đều dương: áp dụng BĐT $AM-GM$ cho $n$ số, ta có:
$$\dfrac{a_1}{a_1+b_1}+\dfrac{a_2}{a_2+b_2}+...+\dfrac{a_n}{a_n+b_n}\ge \dfrac{n\sqrt[n]{a_1a_2...a_n}}{(a_{1}+b_{1})(a_{2}+b_{2})...(a_{n}+b_{n})}\\
\dfrac{b_1}{a_1+b_1}+\dfrac{b_2}{a_2+b_2}+...+\dfrac{b_n}{a_n+b_n}\ge \dfrac{n\sqrt[n]{b_1b_2...b_n}}{(a_{1}+b_{1})(a_{2}+b_{2})...(a_{n}+b_{n})}$$
Cộng vế với vế của 2 BĐT cùng chiều trên, sau vài bước biến đổi ta có ĐPCM. Dấu bằng xảy ra chẳng hạn khi $a_1=a_2=...=a_n;b_1=b_2=...=b_n\ \square$
---
Hai bài trên là hệ quả trực tiếp của BĐT này

mọi người ta tập trung lại 1 tí nhé,mình post bài có kèm theo bài tập là muốn mọi người vận dụng thẳng các kiến thức trên vào bài tập cho vững luôn,dĩ nhiên mình không nề nà gì việc đăng thêm bài mới,nhưng mình nghĩ,việc đó nên để khi ta đã giải quyết trọn vẹn các bài tập
thân
tình hình là ta còn nhiều bài tồn đọng quá
ĐỀ NGHỊ
bài 4 phần 1(kiến thức đã nêu,mong mọi người triệt để áp dụng :wacko:
với mọi a.b không âm,chứng minh
$\sqrt[3]{\frac{a}{b}}+\sqrt[3]{\frac{b}{a}}\leq \sqrt[3]{2(a+b)(\frac{1}{a}+\frac{1}{b})}$

Ô đề nghị này ko ai làm á :ohmy:
---
Áp dụng cái mình c/m đằng trước với $n=3$, ta có:
$$RHS=\sqrt[3]{(1+1)(a+b)(\dfrac{1}{b}+\dfrac{1}{a})}\ge \sqrt[3]{1.a.\dfrac{1}{b}}+\sqrt[3]{1.b.\dfrac{1}{a} }\\ \Rightarrow Q.E.D$$
---
Mn ủng hộ topic đi chứ nhỉ ~O)



#372956 BĐT AM-GM

Đã gửi bởi minhtuyb on 26-11-2012 - 22:57 trong Bất đẳng thức và cực trị

bài nữa nhé
Bài 7: Cho $a,b,c>0$. CMR
$\frac{a^4}{b^2(b+c)}+\frac{b^4}{c^2(c+a)}+\frac{c^4}{a^(a+b)}\geq \frac{a+b+c}{2}$ :icon6:


Đề nghị các bạn giải quyết hết bài tồn động trước khi quăng bài mới :ph34r:
---
$$\dfrac{a^4}{a^2(b+c)}+\dfrac{a}{2}+\dfrac{a}{2}+\dfrac{b+c}{4}\ge 2a$$
Tương tự rồi cộng lại

Do phần này khá rộng nên mình xin trình bày bài tập riêng (hơi nhiều 1 tí) :icon6:
1,Chứng minh với mọi a,b,c $0\leq a,b,c\leq \frac{1}{2}$ thỏa mãn $a+b+c=1$ thì
$\frac{1}{a(2b+2c-1)}+\frac{1}{b(2c+2a-1)}+\frac{1}{c(2a+2b-1)}\geq 27$



Bài này mà $AM-GM$ được thì cũng hơi xoắn :o
---
$$\frac{1}{a(2b+2c-1)}=\dfrac{1}{a(1-2a)}$$
Sau đó ta đi c/m BĐT phụ:
$$\dfrac{1}{a(1-2a)}\ge 27a\\ \Leftrightarrow (3a-1)^2(6a+1)\ge 0$$

2,Chứng minh với mọi a,b,c $\geq \frac{1}{2}$,ta có
$\frac{a^2}{\sqrt{5-2(b+c)}}+\frac{b^2}{\sqrt{5-2(c+a)}}+\frac{c^2}{\sqrt{5-2(a+b)}}\geq 3$


$$\dfrac{a^2}{\sqrt{5-2(b+c)}}\ge \dfrac{(\dfrac{1}{2})^2}{\sqrt{5-2(\dfrac{1}{2}+\dfrac{1}{2})}}=\dfrac{\sqrt{3}}{12}$$
Tương tự rồi cộng lại ta thấy $VT\ge \dfrac{\sqrt{3}}{4}$

3,Chứng minh với mọi a,b,c có $0< a,b,c\leq \frac{1}{3},a^3+b^3+c^3=\frac{3}{64}$ thì
$\frac{1}{1-3a}+\frac{1}{1-3b}+\frac{1}{1-3c}\geq 12$


Tư tưởng tương tự bài $2$, ta xây dựng BĐT phụ:
$$\dfrac{1}{1-3a}\ge 256a^3\\ \Leftrightarrow (4a-1)^2(48a^2+8a+1)\ge 0\ (True)$$



#371298 BĐT AM-GM

Đã gửi bởi minhtuyb on 21-11-2012 - 20:16 trong Bất đẳng thức và cực trị

3,(tổng quát bài 1 lên) với 2n số không âm
$\sqrt[n]{a_{1}a_{2}....a_{n}} +\sqrt[n]{b_{1}b_{2}...b_{n}}\leq \sqrt[n]{(a_{1}+b_{1})(a_{2}+b_{2})...(a_{n}+b_{n})}\(*)$

Nhìn phát thấy ngay là $Holder$ Hình đã gửi
---
-Nếu $\exists a_i+b_i=0\Rightarrow a_i=b_i=0\Rightarrow VT(*)=VP(*)$
-Nếu các biến đều dương: áp dụng BĐT $AM-GM$ cho $n$ số, ta có:
$$\dfrac{a_1}{a_1+b_1}+\dfrac{a_2}{a_2+b_2}+...+\dfrac{a_n}{a_n+b_n}\ge \dfrac{n\sqrt[n]{a_1a_2...a_n}}{(a_{1}+b_{1})(a_{2}+b_{2})...(a_{n}+b_{n})}\\
\dfrac{b_1}{a_1+b_1}+\dfrac{b_2}{a_2+b_2}+...+\dfrac{b_n}{a_n+b_n}\ge \dfrac{n\sqrt[n]{b_1b_2...b_n}}{(a_{1}+b_{1})(a_{2}+b_{2})...(a_{n}+b_{n})}$$
Cộng vế với vế của 2 BĐT cùng chiều trên, sau vài bước biến đổi ta có ĐPCM. Dấu bằng xảy ra chẳng hạn khi $a_1=a_2=...=a_n;b_1=b_2=...=b_n\ \square$
---
Hai bài trên là hệ quả trực tiếp của BĐT này



#372251 BĐT AM-GM

Đã gửi bởi minhtuyb on 24-11-2012 - 22:42 trong Bất đẳng thức và cực trị

*Mình xin dừng phần này ở đây đã
Một số bài tập nho nhỏ cho các bạn
1, chứng minh với mọi a,b,c không âm có $ab+bc+ca+abc=4$ thì
$a+b+c\geq ab+bc+ca$(VMO 1996)
2,Chứng minh với mọi a,b,c dương có $ab+bc+ca+2abc=1$ thì
$2(a+b+c)+1\geq 32abc$

Bài $VMO\ 1996$ thì đã quá quen thuộc rồi mình xin không c/m lại. Còn bài $2$ thì ta cần đánh giá kĩ thuật một chút (xuất phát từ đẳng thức):
---
-Xét $abc=0$ thì BĐT đã cho hiển nhiên đúng
-Xét $abc\ne 0:$
BĐT đã cho tương đương:
$$\dfrac{2(a+b+c)+1}{2abc}\ge 16\\ \Leftrightarrow \dfrac{1}{ab}+\dfrac{1}{bc}+\dfrac{1}{ca}+\dfrac{1}{2abc}\ge 16$$
Đến đây bài toán đã trở nên khá đơn giản, áp dụng BĐT $\sum\limits_{x,y,z,t} \dfrac{1}{x}\ge \dfrac{16}{x+y+z+t}$ với $x,y,z$ dương ta có đpcm
---
Mình nghĩ bài toán này được chế từ đẳng thức $\dfrac{1}{ab}+\dfrac{1}{bc}+\dfrac{1}{ca}+\dfrac{1}{2abc}= \dfrac{2(a+b+c)+1}{2abc}$. Vậy tại sao các bạn ko thử tự chế cho mình vài BĐT khác ? :wub:

P/s: Không nhất thiết là bài nào cũng đổi biến rồi làm trâu nhỉ? :icon6:



#321197 Topic phương trình, hệ phương trình vô tỉ

Đã gửi bởi minhtuyb on 31-05-2012 - 11:28 trong Phương trình, hệ phương trình và bất phương trình

Ờ mà hết bài rồi thì phải. Thôi anh em làm tạm :P:
Bài 59: Giải hệ phương trình sau:
$$(I)\left\{\begin{matrix}\frac{1}{\sqrt{x}}+\sqrt{2-\frac{1}{y}}=2\\ \frac{1}{\sqrt{y}}+\sqrt{2-\frac{1}{x}}=2\end{matrix}\right.$$
------------------
P/s: Anh Khánh edit bài xong thì nhờ mod del cm trên của mình nhé ^_^



#319394 Topic phương trình, hệ phương trình vô tỉ

Đã gửi bởi minhtuyb on 25-05-2012 - 14:31 trong Phương trình, hệ phương trình và bất phương trình

Bài 30: Giải phương trình:

$$\sqrt{x^2-x+19}+\sqrt{7x^2+8x+13}+\sqrt{13x^2+17x+7}=3\sqrt{3}(x+2)$$

@Loãng píc đó ạ :-w



#321412 Topic phương trình, hệ phương trình vô tỉ

Đã gửi bởi minhtuyb on 01-06-2012 - 10:12 trong Phương trình, hệ phương trình và bất phương trình

Bài 60:
Giải hệ phương trình $\left\{\begin{matrix} (x-1)y^{2}+x+y=3& & \\ (y-2)x^{2}+y=x-1& & \end{matrix}\right.$



Bài 1:

$\begin{gathered} \left\{ \begin{gathered} \left( {x - 1} \right)y^2 + x + y = 3 \\ \left( {y - 2} \right)x^2 + y = x + 1 \\ \end{gathered} \right. \\ \Leftrightarrow \left\{ \begin{gathered} \left( {x - 1} \right)y^2 + x - 1 = 2 - y \\ \left( {y - 2} \right)x^2 + y - 2 = x - 1 \\ \end{gathered} \right. \\ \Leftrightarrow \left\{ \begin{gathered} \left( {x - 1} \right)\left( {y^2 + 1} \right) = 2 - y \\ \left( {y - 2} \right)\left( {x^2 + 1} \right) = x - 1 \\ \end{gathered} \right. \\ \end{gathered} $

$\begin{gathered} \Rightarrow \left( {x - 1} \right)\left( {y - 2} \right)\left( {y^2 + 1} \right)\left( {x^2 + 1} \right) = \left( {x - 1} \right)\left( {2 - y} \right) \\ \Leftrightarrow \left( {x - 1} \right)\left( {y - 2} \right)\left[ {\left( {y^2 + 1} \right)\left( {x^2 + 1} \right) + 1} \right] = 0 \\ \Rightarrow \left[ \begin{gathered} x = 1 \Leftrightarrow y = 2 \\ y = 2 \Leftrightarrow x = 1 \\ \end{gathered} \right. \Rightarrow \left( {x;y} \right) = \left( {1;2} \right) \\ \end{gathered} $

-----------Đề thi vào lớp 10 chuyên ĐHKHTN Hà Nội năm học 2011-2012----------



#322186 Topic phương trình, hệ phương trình vô tỉ

Đã gửi bởi minhtuyb on 03-06-2012 - 21:23 trong Phương trình, hệ phương trình và bất phương trình

Rảnh chế 1 bài chơi :P:

Bài 67: Giải hệ:
$$\left\{\begin{matrix}\sqrt{y+1}=\sqrt{2x+y}-1\\ y^2-7x^2+10x-6y-10=0\end{matrix}\right.$$



#319381 Topic phương trình, hệ phương trình vô tỉ

Đã gửi bởi minhtuyb on 25-05-2012 - 14:04 trong Phương trình, hệ phương trình và bất phương trình

Bài 29:
Giải phương trình $$\frac{x^2+6x+15}{x^2+6x+11}=\sqrt{x^2-6x+18}$$

SOLUTION:
$ĐKXĐ: x\in R$
-Pt đã cho tương đương với:
$$1+\frac{4}{(x+3)^2+2}=\sqrt{(x-3)^2+9}(*)$$
-Có: $VT(*)\le 1+\frac{4}{2}=3;VP(*)\ge \sqrt{9}=3\Rightarrow VT(*)\le 3 \le VP(*)$
Theo gt thì BĐT trên xảy ra ở dấu bằng nên:
$$\left\{\begin{matrix}x+3=0\\ x-3=0\end{matrix}\right.\text{Hệ vô nghiệm}$$
Vậy pt đã cho vô nghiệm



#557825 Bất đẳng thức chuẩn bị cho kì thi THPTQG 2015-2016

Đã gửi bởi minhtuyb on 04-05-2015 - 16:33 trong Bất đẳng thức và cực trị

Dạo này thấy topic hơi chìm,góp thêm 1 bài!

Cho $x,y,z$ thực dương thỏa mãn $(x+y)^2+4x^2y^2+1=(2z^2+1)^2$.Tìm GTNN của:

$P=\frac{16x^3}{(y+z)^3}+\frac{16y^3}{(x+z)^3}+3.\frac{xy+1}{z^2+1}$

Từ giả thiết ta có:

$$+) (2z^2+1)^2=(x+y)^2+\frac{(4xy)^2}{4}+1\leq (x+y)^2+\frac{(x+y)^4}{4}+1=\left [ \frac{(x+y)^2}{2}+1 \right ]^2\\ \Rightarrow 2z\leq x+y$$

$$+) (2z^2+1)^2=(2xy+1)^2+(x-y)^2\geq (2xy+1)^2\Rightarrow z^2\geq xy$$

 

Áp dụng bất đẳng thức $\frac{a+c}{b+c}\geq \frac{a}{b}$ với $b>a>0, c\geq 0$, dấu bằng khi $c=0$ với $a=xy,b=z^2,c=1$, ta có: $\frac{xy+1}{z^2+1}\geq \frac{xy}{z^2}$    (*)

 

Áp dụng BĐT Cô si 3 cho số không âm, ta dễ có:

$$\frac{16x^3}{(y+z)^3}\geq \frac{12x}{y+z}-4$$

$$\frac{16y^3}{(x+z)^3}\geq \frac{12y}{x+z}-4$$

 

Từ ba bđt trên suy ra:

$$P\geq 12\left ( \frac{x}{y+z}+\frac{y}{x+z}   \right )+3.\frac{xy}{z^2}-8$$

 

Ta có:

$$(z+x)(z+y)=z^2+xy+yz+zx\leq 2z^2+yz+zx=z(2z+x+y)\leq z(x+y+x+y)=2z(x+y)$$

Nên:

$$\frac{x}{y+z}+\frac{y}{x+z} =\frac{x^2+y^2+z(x+y)}{(z+x)(z+y)}\leq \frac{x^2+y^2+z(x+y)}{2z(x+y)}=\frac{x^2+y^2}{2z(x+y)}+\frac{1}{2}$$

 

Suy ra:

$$P\geq 6\frac{x^2+y^2}{z(x+y)}+3.\frac{xy}{z^2}-2
\\=3\left[  2\frac{x^2+y^2}{z(x+y)}+\frac{xy}{z^2} \right ]-2
\\=3.\frac{2z(x^2+y^2)+xy(x+y)}{z^2(x+y)}-2
\\\geq 3.\frac{2z(x^2+y^2)+xy.2z}{z^2(x+y)}-2
\\=6.\frac{x^2+y^2+xy}{z(x+y)}-2
\\\geq 6.\frac{\frac{3}{4}(x+y)^2}{z(x+y)}-2
\\=\frac{9}{2}\frac{x+y}{z}-2
\\\geq \frac{9}{2}.2-2=7$$

 

Dấu bằng xảy ra khi $x=y=z$

Vậy giá trị nhỏ nhất của biểu thức đã cho là $7$

 

*Nhận xét: Hai khó khăn lớn nhất ở lời giải này là việc xử lý phân thức thứ 3 ( như ở (*) )  và việc làm gọn mẫu số $(z+x)(z+y)$ để xuất hiện hạng tử $z$, từ đó giảm độ phức tạp của bài toán.




#318311 Topic yêu cầu tài liệu THCS

Đã gửi bởi minhtuyb on 21-05-2012 - 19:07 trong Tài liệu - Đề thi

Ai có đề cho em xin cái.


ĐỀ THI TUYỂN SINH VÀO LỚP 10 CHUYÊN QUẢNG BÌNH NĂM HỌC 2010-2011

Search được cái đề vòng 1 thôi:
http://dethi.violet....ntry_id=5807365



#327480 Ảnh thành viên

Đã gửi bởi minhtuyb on 21-06-2012 - 08:09 trong Góc giao lưu

sao nhỏ con vậy tú , :wacko: :mellow:

Gì? hơn 6 yến với ~1m7 mà nhỏ con :wacko:

Mặt Tú có cái nét gì đó đẹp trai giống tui Hình đã gửi

Gặp ai ông cũng nói vậy thì phải >:)



#327456 Ảnh thành viên

Đã gửi bởi minhtuyb on 20-06-2012 - 23:03 trong Góc giao lưu

Có ai nhận ra khung cảnh đó ở đâu không?

Chắc ở trường nào đó hả thầy :P
P/s: Ai có ảnh mình chưa :-?



#325676 Ảnh thành viên

Đã gửi bởi minhtuyb on 15-06-2012 - 22:55 trong Góc giao lưu

Mấy chú nhà ta vỡ mộng rồi! Khổ!

Khó đỡ thật, X-girl xuất hiệnHình đã gửi
----
P/s: Đi treo lại cái avatar con bạn cho đỡ tủiHình đã gửi



#327462 Ảnh thành viên

Đã gửi bởi minhtuyb on 20-06-2012 - 23:32 trong Góc giao lưu

ko biết mọi người sao nhưng t chưa có :P

Tú úp lên đi :D

Ok, để duy trì ý nghĩa "nhân đạo" và "nhân sinh" của topic xin phép mọi người up vài tấm hình :P:
Hình đã gửi
------------------
Hình đã gửi
----------
Hình đã gửi
--------------
Đã chuẩn bị sẵn bao hứng gạch >:)



#327594 Ảnh thành viên

Đã gửi bởi minhtuyb on 21-06-2012 - 13:08 trong Góc giao lưu

ke3, thế a post ảnh lên phát cho bọn e thưởng thức ^^

Ảnh anh bí bòi có trong topic này rồi mà :P

Chả thấy đứa nào hơn mình nản!

Em nè anh Hình đã gửi

Do em đẹp trai nên ăn gạch hoài anh à Hình đã gửi
ÔNG TRỜI ƠI, CHO CON ĐẸP TRAI LÀM GÌ CHO CON ĂN GẠCH HOÀI VẦY Hình đã gửi

Thêm 1 cục vì tội "đẹp" trai mà thích tự sướng Hình đã gửi
-----------------
P/s: Giờ mới biết box này ở chế độ "No posting" Hình đã gửi



#328751 Ảnh thành viên

Đã gửi bởi minhtuyb on 24-06-2012 - 19:20 trong Góc giao lưu

Tin hót đây !!! Siêu nóng luôn :

Đây là ảnh anh Nguyễn Công Định (ongtroi) vừa đi giả phật kiếm tiền về đấy !!!


P/s: Chém mạnh tay vào !!!

Thể loại gì đây trời Hình đã gửiHình đã gửiHình đã gửi



#327610 Ảnh thành viên

Đã gửi bởi minhtuyb on 21-06-2012 - 13:57 trong Góc giao lưu

Nơi nào có thánh, nơi đó có gạch Hình đã gửi
Hình đã gửi



#328952 Ảnh thành viên

Đã gửi bởi minhtuyb on 25-06-2012 - 10:11 trong Góc giao lưu

Áo trắng em à Hình đã gửi

OMG! Đau mắt quáHình đã gửi